Hacker News new | past | comments | ask | show | jobs | submit login
LK-99: Team of Southeast University observed zero resistance below 110 K (twitter.com/lipez400)
640 points by thecopy on Aug 2, 2023 | hide | past | favorite | 341 comments




To me this is encouraging. 110K is really high! It's higher than YBCO which is what they make commercial "high temperature" superconducting tape out of. IMO if this is legit (I have no way of judging the source) then this is the first good third party evidence that the original researchers are not just doing some kind of fraud. A total fraud wouldn't have actually discovered a novel high-temperature, but not room temperature, superconductor.

At worst it seems to me like they discovered a cool new superconductor that may have commercial applications. At best, they may have a variant of this material that really is a room temperature superconductor. Either way, there is an important scientific discovery here.

I'm surprised the prediction markets don't seem to be reacting. Maybe the source isn't actually credible? Any Chinese speakers here know?


They aren't reacting because this isn't really proof of anything. The test apparatus they used can't actually measure 0 resistance. Superconductivity is implied (thats the most straightforward explanation) but not actually demonstrated here. Its very hard to interpret from the limited data presented. There could be a whole host of contaminants, issue with the experimental setup etc etc. Its hard to go from look at the interesting material properties in this graph to room temperature superconductor. This is also the only confirmatory information we have from the team; their sample didn't show diamagnetism.

The reasons to be skeptical are: 1) lot of videos floating around that are just at the threshold of convincing 2) poor results from other teams 3) there is a history of superconductivity discoveries like this that never pan out 4) skepticism of the original paper


We are in the fog of war phase state of this discovery process so there are data points in every direction with the kpop X-itter drama and sensationalism thrown in to the spin for good measure. Hard to tell if this is geek meme stock phenomenon meets the Reddit Boston Bombers hunt or just the real-time globally hyper connected social media version of exactly what would have happened if social media and arXiv were around in Christmas 1938 when two German chemists noticed a flash and detected Krypton where it should not have been and the Nuclear Age began.


> The test apparatus they used can't actually measure 0 resistance.

Can you explain how you would make that measurement? It seems like your testing equipment would need to be made of superconductors as well, right?


Usually superconductor resistance is measured using four probes: current goes through the two outer probes, and voltage is measured across the two inner probes. Then V=IR.

Lead resistance can play a role, but superconductivity is a phase transition: there's a significant discontinuity. It's not a matter of something going from 0 + e (for small e) to 0 (which doesn't happen), but one of going from x (x >>> 0) to 0. When that phase transition happens, it's obvious.


Noob question: how do you know you're not going from x >>> 0 to 0 + e?

Not clear to me how you know the resistance is nothing at all vs just very small.


I think most people are just seeing the headline 110K instead of realizing the implications (not a scam, not a mistake, hard to replicate).

I was quite sceptical earlier today due to unconvincing videos but this finding pushes it heavily into "This might be it" territory.


From my perspective the story has went like this:

1. It's probably fake/fraud/error/hopium.

2. Yeah but, it's not a superconductor, it's just diamagnetic.

3. Yeah but, theoretically maybe, but not practically.

4. Yeah but, it's just a warmer superconductor, not room temperature.

I don't know if my sensibilities are "bayesian" or something, but while the naysayers are totally correct, their predictions are not where the trend is heading. This thing is consistently defying expectations and gradually becoming more "important" as more information comes out.

I believe if the Q-Centre team say it's room-temperature SC, then given the evidence so far I expect to get closer to that conclusion over time. If it fails, then I think it'll fail for very good reasons, not simple error or incompetence. At this point, fraud is totally ruled out.


Wasn't fraud basically always ruled out?

If you want to benefit from dubious and/or fake research, you'll have to be a lot more subtle about it.

Announcing you just found the holy grail will get you all the attention in the world, attention you don't want if you just forged the paper to advance your career.


I'm still unclear. Why is the test limit 110K? If this is supposed to be usable at ambient temperature then this seems like not a useful finding. Is it just some artifact of how superconductivity is measured?


Some people say they can get it to work at room temperature, and others say they can't.

Testing a sample at superconductivity-friendly temperatures helps figure out whether there's something to investigate here, since most materials do not superconduct at any useful temperature.


Most materials are not superconductors at 110K. If this one is, it is possible Koreans did not fake room temperature results but rather their sample is slightly different for some reason, maybe pure luck, maybe some part of process that they didn't clearly define or that their precursor materials have certain impurities.


It is apparently extremely difficult to fabricate LK-99 and most of the time the process being used creates similar substances but without appropriate Cu substitution in its lattice. All of the videos have been of generally quite small samples and the manufacturing difficulty is the purported reason.

The SK also supposedly had more details on this but with the leak of the paper those haven't been cleaned up and added to the paper.


I thought the original hype around LK-99 was that it was supposed to be easy to manufacture with materials most labs have on hand? E.g. https://news.ycombinator.com/item?id=36865106


Amusingly, some friends asked me about it shortly after the original announcement. I'm not a superconductor person, but I am a materials person.

My response was, basically, the biggest risk (after toxicity) "is that they got lucky and there are subtle things they did that aren't in the recipe which will turn out to have been critical."

I've done a lot of materials research. It's pretty likely that they got lucky on a small percentage of the samples and spent years trying to figure out what on earth was different. I am excited that 10,000 other labs will stand a very good chance of finding out.


Exactly my take. The big question for me is whether or not they haven't accidentally included some contamination that makes all the difference or whether or not the geometry of the placement of the atoms is controlled tightly enough and whether either of those is going to make a huge difference in the outcome. And then there are a hundred different process errors that could have similar effects. This could take a while to be nailed down.



- It was naive to think a simple recipe was going to play out with everyone getting what they wanted easily when they tried to follow it.

- It's still a huge deal if the manufacturing is as easy as it appears to be, even if most of the replication attempts aren't getting _quite_ the right thing. It's hilarious if room temperature superconductors were always "Put these two things in a lucifer furnace and roll a die" away.


> It's hilarious if room temperature superconductors were always "Put these two things in a lucifer furnace and roll a die" away.

Is it, though? I mean it's hilarious sure but if we were to draw parallels to the mother of all elements, the celestial furnaces in the sky, it seems less wild.

A pretty lucky roll of the die nonetheless.


It's extraordinary how many replication experiments are happening so quickly and how tantalizing the results are. It would be optimistic to say the original research was just going to unlock this tech for everybody at once but even if it's the clue that leads to dozens of refinements that eventually lands on a winning material then it's still a landmark event.


The materials are easily on hand. That doesn’t guarantee it was easy to manufacture.

It’s not made of out of exotic elements, mostly just lead and copper. So in theory anyone can make it, but it’s just hard to do it correctly.


The consensus of this forum was absolutely that it's super easy to manufacture, any chemistry youtuber can do it, and that we are going to know whether it works or not within 48 hours.

I wonder if the people who steered the discussion just pretended to be experts or what happened. Where were the people who now in these threads explain the reasons why it is really difficult to bake? Or are those the same users?


Probably people who knew above average about the process and sourcing materials to realize you just had to throw some common stuff in a furnace but without being experts in material science. Even then if it's tricky to do its most likely because the process is not well know/refined yet


The other day, about a third of top posts on hn where about this. I got the impression there was an organised campaign by the believers. The usual healthy skepticism was buried beneath the cheerleaders. Most top comments where along the lines "I never believed it. Until now".


The ingredients and tools aren't exotic, but chemistry is still a pain in the ass.


The common understanding seems to be that producing true LK99 is very difficult as you need to get the crystal structure just right. Many samples probably have mixes of the correct and other structures. That's why some float and some don't. The sample might be quite "dirty", meaning the critical temperature is lower.


Seems like it may depend on getting specific binding sites.

> Finally, the calculations presented here suggest that Cu substitution on the appropriate (Pb(1)) site displays many key characteristics for high-TC superconductivity, namely a particularly flat isolated d-manifold, and the potential presence of fluctuating magnetism, charge and phonons. However, substitution on the other Pb(2) does not appear to have such sought-after properties, despite being the lower-energy substitution site. This result hints to the synthesis challenge in obtaining Cu substituted on the appropriate site for obtaining a bulk superconducting sample. [1]

1. https://arxiv.org/abs/2307.16892


I believe 110K is where it hit zero resistivity (the lower bound) but any increase from there and it was no longer presenting superconductivity.

If we are to trust the SK team, it's possible the design in the paper is outdated and in-house they have dialed it in to something stable at room temperature.


For non-Chinese speakers: one can jump to 01:50 and 02:32 of the original video, the key figures (XRD and resistance) are presented with English captions.

The data look legit although there is a curious dip in resistance in Fig.3(a) between 200K and 250K. Fig.3(b) is also a bit weird as somehow the resistance behaves irregularly with magnetic field strength.


> I'm surprised the prediction markets don't seem to be reacting.

Which way would you have the prediction markets go? Is this evidence for or against room-temperature superconductivity?


Since IMO this makes fraud a less likely explanation, and it seems easily possible to me that there are many materials in this family with different critical temperatures, and it seems really unlikely that the original researchers would correctly measure superconductivity but simply get the temperature wrong by more than 200K, then it seems strongly positive to the probability that the original researchers discovered room temperature superconductivity. Not confirmation, of course.


What's impressive to me is how simple making it is. There are still low hanging scientific discoveries that are out there but that we haven't figured out yet.


Like ancient Roman concrete.


I believe that one was recently figured out - https://news.mit.edu/2023/roman-concrete-durability-lime-cas....


>At worst it seems to me like they discovered a cool new superconductor that may have commercial applications. At best, they may have a variant of this material that really is a room temperature superconductor. Either way, there is an important scientific discovery here.

I feel like this whole thing is going to be graphene all over again. A massive initial hype around "world changing tech" that ends up being very difficult and costly to scale and produce commercially.

Probably we'll see some niche applications in industry after a decade of further refinement, but I'm not holding my breath for hoverboards any time soon.


> I feel like this whole thing is going to be graphene all over again. A massive initial hype around "world changing tech" that ends up being very difficult and costly to scale and produce commercially.

That's kinda how science goes. The first free-electron laser was built in 1971. It took decades to develop and commercialize free-electron lasers and we still in the early days.

It is rare for a single discovery to revolutionize everything especially in the 21st century. Typically it is a series of discoveries in a row that taken together allow new processes and technologies. Each discovery on that road is important.

The short term impacts of discoveries are overestimated, the long term impacts are underestimated.

On a 50-100 year time scale, the perception of graphene may be very different from what it is now. We are looking at a tadpole and saying, I thought these eggs were suppose to hatch into frogs, not tadpoles.


The history of the laser makes for fascinating reading. Moreso because the longer they looked the harder it was to find things that were candidates that they couldn't get to exhibit the effect given the right conditions, but some of those were so extreme that they would have been discarded outright in the early days.

The idea originated with Einstein in the middle of WWI, and it took until 1960 for the first working device to be made with lots of incremental progress in the middle. After that it's been a non-stop series of inventions each of which gave rise to new practical applications and new avenues of search. Originally described as 'a solution looking for a problem' it turned out that indeed, it was the solution to a lot of problems, we just didn't realize how applicable this tech would be.


It’s worth pointing out that graphene literally is just a single layer of graphite. Making macroscopic objects out of graphene would mean layering them. So effectively we already have graphene in the form of graphite fiber (also called “carbon fiber”).

Graphite with zero defects and perfect crystallinity has the same properties as graphene. It IS layers of graphene.

(This is all essentially true for carbon nanotubes, too, which are just tubes made of graphene… maybe graphene is more chemically reactive due to the ends of the sheets though.)


I think the rule of thumb, though, is that most things don't turn out to be viable, and then most viable things are only barely useful due to constraints. But occasionally they DO occasionally come up with world-changing technology.

Dark example, but the atomic bomb for instance was a dramatic improvement in our capacity to destroy things: night and day difference.


My guess would be that it'd be similar to silicon wafer production and just require lots work to industrialize the process and hone production. Graphene is just particularly difficult due to its 2-d nature, but lk-99 looks to a regular crystalline structure.


What prediction markets are you looking at? Are there any concrete examples of these markets being accurate, or just a venue for gambling addicts to feel smart?


> Are there any concrete examples of these markets being accurate

There are prediction markets for US elections, which tend to be pretty good. In general you should only expect markets with high liquidity to be very hard to beat, and the US elections have relatively low liquidity, so IIRC Nate Silver still outperforms them (but only by a small amount).

The reason you should expect high-liquidity markets to be hard to beat is that, if they were easy to beat, you could easily get rich by betting on them. So the common-sense idea of "there's no easy way to get rich" implies that they cannot be easy to beat.

The main exception to this principle is when there's a huge amount of money that intentionally places "bad" bets on the market, usually because they want to hedge against something. (e.g. I don't want this to happen, so I'll bet that it will happen, that way at least if it does happen I'll make some money.) This is why the TIPS spread, which can be interpreted as a prediction market for US inflation, consistently over-predicts inflation. Entities that stand to lose from high inflation use TIPS to hedge, which causes the TIPS spread to be too high. This could be corrected with better market design, but the treasury hasn't done that yet.


One prediction market company spent a day spamming e v e r y tweet with a link to their own stuff, so it's adopted a meme-y link to LK-99 but it's honestly unrelated other than a clever marketing campaign


My question is also broader than LK-99, and my point still stands.


And my reply still stands, if not read for signs of an adversary, you would read a comment that reinforces (B) of the (A) and (B) you offered people to reply with. Its for gambling addicts


If it's for gambling addicts who know very little then it shouldn't be that difficult to make money on it (not necessarily on LK99, but in general).

Either the markets have more insight (swarm intelligence or agents who know more leveraging their knowledge) than me – and in that case they give me some information – or they have less insight and in that case I should play them.


Does anyone know which real money prediction market is deepest? The ones I see everyone referencing are fake money



You're reply falls like a house of brick with no mortars.


YBCO superconducts as high as about 93K, so we're talking 17K difference. Which is useful but not wildly so. LN2 boils at 77K, for reference.


YBCO is used in cutting edge Tokmak fusion reactors like SPARC. I'm not sure how an 18% increase in allowable temperature would change how much closer you could move the magnets to the plasma. Squinting at Figure 1 in [0], it seems possible you could get significant increased in Q from 18% increase in temperature.

[0]: Overview of the SPARC tokamak. Journal of Plasma Physics https://www.cambridge.org/core/journals/journal-of-plasma-ph...


for YBCO the critical field increases the further you get from its threshold temp.

If being further from the threshold temp implies a higher critical field then that is very useful.


Mercury based cuprate superconductors go even higher than YBCO, like 145K at ambient pressure.


> I'm surprised the prediction markets don't seem to be reacting.

Where can we look at prediction markets on this? For those of us who aren't familiar.

I'm very curious -- because unlike sports scores or political elections, this seems like such a hard thing to define a prediction market around, because what is the exact threshold you're defining and what is the threshold of proof of that thing and on what date is that decided?


Those rules are defined in the actual bet. I think it was "three replications of room temperature superconductivity by X date" or something like that.

I looked closely because I very much wanted to bet tens of thousands of dollars on "no", but I couldn't because I'm in the US. The two outcomes would be I'd make a lot of money or humanity would make the biggest breakthrough in generations.


> I think it was "three replications of room temperature superconductivity by X date" or something like that.

That doesn’t sound like a sufficient market resolution condition. Surely it would need to be something like “declared by specific Party X to be a room-temperature superconductor” where Party X is sufficiently trusted by all market participants.


Manifold is a funny money market, polymarket is the only one with any semblance of volume in my experience. https://polymarket.com/event/is-the-room-temp-superconductor... resolution criteria also leaves a bit to be desired imo


I was worried about that too before making the bet, but I realized I couldn't do it anyway and stopped worrying about it.

You can find the whole resolution criteria here if you want:

https://manifold.markets/QuantumObserver/will-the-lk99-room-...


Thanks. And wow, that's as fuzzy as I was afraid it would be:

> ...Willing to adjust this criterion after receiving more info from relevant theorists/experimentalists...

> ...I don't intend to require that replications be published in a peer-reviewed journal... However, I do intend to wait a few weeks/months to resolve so that any pre-print can be adequately investigated...

> ...Since high Tc superconductivity is not my specific field of expertise, I'm willing to defer to a consensus of subject matter experts on whether a pre-print is convincing or not, and I am willing to contact some beyond the usual twitter personalities...

In other words, if there's any kind of gray area in the results, it's going to be whatever this person decides, whenever they want to decide it. Definitely not something I would ever put money behind.


Yeah, I was definitely concerned about losing my money to a technicality.


Yesterday someone posted a couple links and I remember this one:

https://manifold.markets/QuantumObserver/will-the-lk99-room-...


Metaculus has some markets on LK-99, for example



The initial claim was speculated to be room temp and ambient pressure superconduction. 110K is a far cry from that.


Everyone agree. Oversimplifying, there are 3 categories to classify the initial team:

1) A bunch of clowns.

2) Interesting

3) Next Nobel prize winners

I think they had a good reputation in the community, so the opinion of the hivemind was to discard 1. But there was still the possibility of a honest mistake or something weird.

If this post is correct, other team confirmed that they discovered a new family of "high temperature" superconductor. The old families have been tweaked and explored to death. A new family gives a lot of room to optimize the composition and building process. So perhaps they found it.

If this is confirmed they are definitively in the "interesting" category, at least.


I disagree. First, there have been many RESPECTED scientists who have thrown it all away for fraud. This happened RECENTLY with retracted papers on near-ambient superconductors so you should be wary. Just like cold fusion claims. So it’s possible at least one of these did, too. Another possibility is they fooled themselves (see Feynman’s quote) and maybe one of them faked data to help garner credit.

Secondly, there is no middle ground here. If it’s a room temperature superconductor like some of their data showed, then it’s Nobel Prize.


From my understanding the first paper was published without authorization. So it was likely they were not ready for it to be published. So 2) still seems possible to me. Like what if the team was it seems like we have a room temperature super conductor we need to do more work to be 100% sure though, and then someone then just goes and publishes your rough draft. It could still just be something interesting, and not fraud.

-EDIT- Like if the first paper was not released as is let's say after their testing it turns out to not be room temperature and ambient pressure. They could have easily revised their paper to be about new class of super conducting material ect.... or just not published the paper if it was dud.


I agree that 1) is not discarded, it's never discarded. From scam to honest mistakes, sloppy handling, weird unexpected things, ... I'm just saying that if I has to guess, it's probably not 1).

There is middle ground. They may have discovered a new kind of (not room temperature) superconductor that is an achievement. It's good to publish a few papers, but we would not be discussing it all week.

Another middle ground is it is a superconductor at -15°C (5°F). That's not room temperature unless you live in Antarctica and forgot to close the window. But it's freezer temperature that is much cheaper than liquid nitrogen or liquid helium.


> Another middle ground is it is a superconductor at -15°C (5°F). But it's freezer temperature that is much cheaper than liquid nitrogen or liquid helium.

I suspect this will make MRI machines much cheaper than they are currently and will radically improve the healthcare for many.

You got me thinking about the viability of superconductor-based EV motors that are cooled by compressed gas refrigeration; drink chillers might come standard on EVs since you'll be having beefier compressors than for just climate-control


It’s easy to make a high tc superconductor have lower tc through poor production quality. It may very well be that a poor sample is 110K while a high quality sample is >400K.


Looks like 110K still puts this in the top 5 highest temp known superconductors and it only takes a couple of days to produce out of the blue.

So even if that's all it sounds like an important discovery.


110K actually makes me excited as someone with a physics background. It seems more realistic, and there's always the possibility of finding full room temp later, too.


That could still be possible. We haven't found the ideal form of LK-99 yet, so another lab might have a form which is only stable to 110K, but that doesn't mean all will see the same thing.


Not to mention that every new category of SC discovered helps nail down the theory of how it all works.


[flagged]


Spicy take:

Theory is shit (betraying my bias here). If someone released a paper that theoretically proved a material was a superconductor from first principles, I would ignore it, every bit as much as I would if it had the opposite results. In materials, experiment has always led theory. Theory has use in suggesting new angles to explore with experiment, but theory is fundamentally reactive: get new data, try to fit that data using known principles and math. It's simply not equipped to say whether reality is real or not.

Honestly, I think anyone who thought the DFT-based papers mattered at all doesn't really get how science works. The politics here is also worthy of keeping in mind: if a theorist comes out and says something is flat out impossible, he/she gets egg on their face if it turns out to be possible. If someone says it's possible, there's plenty of wiggle room to justify yourself if the material doesn't pan out (plus, if it does, you get lots of credit for offering the first theoretical explanation).


>Honestly, I think anyone who thought the DFT-based papers mattered at all doesn't really get how science works.

It's a very spicy take to say DFT-based papers don't matter at all. Anyone who thinks they conclusively prove or even provide particularly strong evidence that LK99 IS a RTAPS or a superconductor at all is misunderstanding, of course, but there's a lot of room between "doesn't matter at all" and "doesn't provide strong or conclusive evidence"

They propose some theoretical ways in which LK99 COULD be an RTAPS. If we couldn't even come up with theoretical ways it could do so, then that is obviously a bad thing for the idea that LK99 could be a superconductor.

I agree with your general sentiment, just not the level of it.


Naw, theory is perfect as long as there's the math to back it up, which in this case there isn't.

There are plenty of theories backed up by hard math which can be summarised as "this _should_ work" and then it usually does.

As long as it's classical physics I guess. Even with math it seems quantum physics still surprises us.


I have little to add other than I f---ing love your spicy take :D

...and agree with most of it. Yeah ofc I know results lead theory in all physics esp materials... :)

Disagree re: the not getting how science works, though you might be facetious?... the fact that it just so happens that if you plug the numbers obtained by experimentation into a current reasonable model, HUH, SOMETHING INTERESTING! That's huge. No, I don't think that's a non-result, and (intentionally using the "appeal to authority" logical fallacy in place of a real argument) Dr. Derek Lowe agrees, so we know how science works. ;-)

Can you expand more about what you mean by "the politics here"? (your sentence 1-5 I couldn't agree more with)


> Disagree re: the not getting how science works, though you might be facetious?

Mostly tipsy facetiousness. I mostly wanted to mock the people who thought the DFT analysis was strong evidence that LK-99 is actually a superconductor. Of course, real-life theorists (and experimentalists) know that the advance of science is a progressive dialog between the two groups.

> Can you expand more about what you mean by "the politics here"?

Politics in the sense of getting (positive) recognition. It's absolutely possible for someone to use theory to hypothesize whether LK-99 is a superconductor (RTP or otherwise), and that attempt provides useful predictive value. But someone posting a negative result from that faces more risks than a positive result: the positive case always has an escape hatch (e.g. other factor wasn't accounted for). Experimentalists are lucky, in that escape hatches--e.g. the exact method to reproduce wasn't known to us--are always available to the negative case, because experiments are so messy.


> a Chinese or Russian lab claiming something and posting to twitter has very little value

Why the national specificity here ?


Prejudice


Saw it on Fox News!


Experience.


nm.


You know why. The guy probably rants about the ccp or putin every day. Sound like anyone you know? I wish dang would do better handling the obvious rampant flamebait.

@dang flamebait alert. Please climb up thread to find the offending party.


This is my first comment on this material, because I've been skeptical, but this result shouldn't be understated. It basically confirms the existence of a new family of high-temperature superconductors, the first of its kind since the cuprates in 1986. And for context, in the initial paper on the cuprates the critical temperature was measured to be 36 K. By synthesising other members of the cuprate family and optimising the growth, it was possible to raise it to 127 K in the subsequent few years. The 110 K seen today for LK-99 sets a baseline, and it's a very good baseline at that. Confirm me as excited.


> It basically confirms the existence of a new family of high-temperature superconductors, the first of its kind since the cuprates in 1986.

Specifically, it seems, a family based on a novel superconductivity mechanism. If this a real effect, there's not only significant scope to improve LK-99 synthesis, but also scope to find new materials that could use the same mechanism that operate at even higher temperatures, are easier or cheaper to produce, or that can move more current.


One of the DFT papers suggested gold or silver for doping agents and, I’m no expert in the field but I feel like there’s a ton of room to explore the doping process of this material.


The jump at 250K looks like bad contacts to me that fixed themselves during cooldown. Rewiring the sample and remeasuring should get rid of it. The field dependence is indeed a bit weird. One would expect the critical temperature to reduce with increasing magnetic field (magnetic field weakens superconductivity), but here you don't see that. It's expected that the superconductivity is very strong for such a high critical temperature and the fields applied are not strong enough, but with better measurements you should still see a small reduction in the critical temperature.


My intuition tells me that the theory here will be more interesting than the current application, LK99. If it pans out then thousands of brilliant minds will iterate on the theory to expand it and to refine applications of it.


Huge news. This means it is not a scam as a discovery of this type of superconductor would be big enough.

It also means it is unlikely they were measuring wrong for multiple years.

My layman take is that Korean sample is just a better or different batch that is properly superconducting at room temperature.


As a layman, how do you know this is huge?


Assuming they discovered a low cost, room temperature, room pressure, superconductor, there are many HUGE technological advancements that can be made that would impact your daily life.

Possibilities include improved battery longevity in all devices(probably in an order of magnitude), low friction transport improvements (ie. cheaper high speed rail), and faster and higher bandwidth wired connections.

https://en.wikipedia.org/wiki/Technological_applications_of_...


> improved battery longevity

No

> probably in an order of magnitude

Absolutely not.

> Low friction transport improvements

This material is superconductive at 110K (-163C). Not exactly usable for transport applications.

> faster and higher bandwidth wired connections.

Absolutely not, resistance has no impact on bandwidth.

I've seen variations of this comment on hacker news. Superconductors are not magic dust to make things better. They are conductors with 0 resistance. There are certainly applications for that (see the wiki you linked) but like all things based in reality those are all a lot more muted and probably not possible with the current materials.

You are getting excited about the possibility of wires. There are certainly cool things you can do with a nice wire, but it's still a wire. You can't store power much with it, It's too big to make logic circuits with, and applications (like levitating a train) require too many amps for our poor wire to remain a special wire. (Most super conductive materials lose conductivity when amps are too high).


>and applications (like levitating a train) require too many amps for our poor wire to remain a special wire. (Most super conductive materials lose conductivity when amps are too high).

I was wondering if there was a current limit on superconductors.

1) Is there any understanding as to why superconductivity breaks down at higher amperage? 2) If so, is there any explanation as to why that doesn't require a PhD in physics?


> Is there any understanding as to why superconductivity breaks down at higher amperage?

This is a good read [1]

> As long as the induced magnetic field at the edges is less than the critical field, the material remains superconducting, but at higher currents, the field becomes too strong and the superconducting state is lost. This limit on current density has important practical implications in applications of superconducting materials – despite zero resistance they cannot carry unlimited quantities of electric power.

Tl;Dr (and probably wrong) as current flows through any conductor it creates a magnetic field. In superconductors when that magnetic field gets too strong it impedes current from being able to flow. A little like a traffic wave [2]. Everything works fine so long as there's enough space between cars to allow for them to speed up and slow down, but as the density of the cars increases if someone slows down that has a reverberating effect down the chain.

The magnetic field on a superconductor in turn induces a current on the conductor in the opposite direction.

Here's a video discussing some of the implications of this effect in a way that seems counter intuitive :) [3]

[1] https://en.wikipedia.org/wiki/Critical_field

[2] https://en.wikipedia.org/wiki/Traffic_wave

[3] https://www.youtube.com/watch?v=oI_X2cMHNe0


as always the smart comment, ignore ant, them all


No I mean why is this "huge" when each time that's happened it's been deflated within 12 hours:

- initial LK-99 paper upload on Arxiv: HUGE, then it's probably nothing until replication (waiting)

- DFT release: HUGE, then probably nothing (DFT has poor predictive power)

- 110K SC: HUGE, then ... ?

In every case it's been the laymen saying HUGE, then the experts saying it's probably insignificant. Then the laymen settle on what the experts said.


Derek Lowe is a layman? I grant his specialism doesn't precisely match the field, but it's easily close enough I'd expect him to be able to smell bullshit on this if there was any, and his latest "In the Pipeline" suggests much more the scent of roses.


Derek Lowe is saying 'this is maybe possible', commentators on Hacker News are saying 'this is practically a done deal, and here are the implications'


Just temper your expectations a bit and don't get hyped up by people on the internet. All of these are exciting developments, but no one is going to know the truth of it until you have multiple independent confirmations in one direction or another. No single piece of evidence is going to be convincing unless it comes from one of the big labs, and they're unlikely to publish quickly because they're going to wait until they have to k solid proof.

As someone with experience working on superconductors, the DFT results and this paper are exciting because they show that at the very least this is likely a new class of superconducting materials at least as good as the current industrial ones. Knowing that the authors are on to something and that the initial claims aren't totally nuts is exciting and fun to post about, but it'll take time to be sure about any of this.


As someone who has worked on superconductivity I'd say that all of these are all potentially HUGE, but meaningless individually because they require experimental replication. They point to at minimum a new class of high temperature superconductors at least as good as current industrial ones. To know if it's truly transformative or not though we'll need multiple confirmations from big name labs. That's going to take time, so this trickle is exciting but won't mean much until the dust settles.


[flagged]


It is not that they are throwing s** at it, but rather that none of you have actually explained why this is actually "huge," as you have been saying multiple times but without actually providing any concrete examples. "It is huge, trust me bro," isn't it.


Until today main hypothesis was that it is a scam by original team or a mistake when measuring resistance.

Now it looks unlikely as it would be very strange to luck into a new superconductor (pretty "good" one too) if they were faking it. It also means it is unlikely they made fundamental measuring mistakes such as thinking the sample is in room temperature when it was at 100K.

What seem plausible is that the process to make the material is not well defined and that there is high degree of variability. Even this chinese 110K replication only one of 6 samples shows superconductivity, meaning there is much room for improvement, perhaps with fine tuning they will find sample with characteristics that Korean team observed.


I don’t know why people say it’s huge, so far all the images released show it is tiny.


Some potential applications for disruption:

Sub-sea cables that are the size of current fiber-optic bundles, which can transmit terawatts of energy with minimal/no losses. The bottom of the ocean is, ironically, at much higher pressure than atmosphere and would actually help increase the tolerances for superconductivity. This means: areas that have abundant power resources can export it with minimal infrastructure costs. Installing the transmission lines in this manner would be orders of magnitude cheaper than current high voltage DC transmission, and could likely bridge entire continents together. The #1 hold back on offshore wind is getting the energy from the farm to the onshore landing point. The energy loss and step-up/step-down transformers, with maintenance on those in kind, can be 30-40% of the project cost. You can also eliminate expensive and inefficient transformers on both ends since you can leave the voltage at generation-levels vs stepping it up to hundreds of thousands of volts in order to transmit it, which adds a lot of complexity. Under-utilized Hydroelectric capacity in northern Quebec could power Southern US states.

The carry on from this in terms of the reduction in required infrastructure for power transmission and delivery is massive. Think of all the copper and aluminum required today to build huge transformers, step up and step down electricity, and get it from the generation plants to your home or business. You could effectively power an entire household on a cable the size of a fishing line compared to cables the size of a sharpie.

The same applies to electronics - if there is a way to use this superconductor in transistors and microchips - the heat loss from operation could be reduced to nil, meaning you can have a chip with little/no thermal loss while operating. This eliminates the need for expensive cooling (again, typically copper or aluminum) and also all the complexity/cost associated with that. The power consumption of these chips is typically a result of the electricity losses as the current is driven into the chip at low voltage. Less thermal waste means much higher efficiency chips, which means less power required to operate them, which means much less consumption (and battery required to supply it). Mobile phones built with superconducting components and chips could last weeks on a standard battery since almost all the power consumption would be the radio, speakers and the screen.

Since the superconducting temperature claimed by the initial team (127 °C) is much higher than most ambient temperatures, this means the potential applications are essentially anywhere outside of a heat source.

Batteries and battery packs could be miniaturized to some extent - reducing transmission losses and eliminating heat on low voltage power means you can reduce the amount of copper required in a battery pack, and inside the battery itself, by a significant margin. This leads to lighter batteries out of the gate. Coupled with lighter cables to carry the power and lighter/cooler electronics to manage and distribute that power and manage the heat - the weight savings could be very significant.

The potential for miniaturization and displacement of heavy/expensive/bulky traditional conductors is very large and not possible to understate.


Certainly no expert here but if you take the original video and replications at face value it does exhibit strong diamagnetism at room temperature. Superconductors are diamagnetic due to the Meissner effect at their superconducting temperature. This would mean LK99 is a superconductor at low temperature and separately be strongly diamagnetic at room temperature (no Meissner effect if it's not super conducting). Would that be unusual?


It's all unusual because it is uncharted territory. My guess would be assuming that it all pans out that this is due to various impurities and that it will take a while before we have a sample that is pure enough that all of these properties can be nailed down for good.

What's strange is that drop at higher temps, that's either a measurement anomaly or something really odd.


> due to various impurities and that it will take a while before we have a sample that is pure enough that all of these properties can be nailed down for good.

Do you think it will be "purity" or understanding material variance/specific impurity?

It reminds me of Fogbank, the nuke material claimed to be "so secret they forgot how to make it." Part of the story of manufacturing difficulty was due to increased purity of modern materials/processes.

In a bizarre twist, the new production facility and reverse-engineered production process yielded a version of Fogbank that was of a higher purity than it had been in the past, according to the article. The problem, however, was that for Fogbank to work as intended in existing warhead designs, that previous level of impurity was actually essential. NNSA had to revise the process to ensure the final product was just as impure.

https://www.thedrive.com/the-war-zone/32867/fogbank-is-myste...


It reminds me of Max Gergel's Isopropyl bromide book:

When Hildebrand ran the baths the silver came out bright, and stuck to the objects; the minute he left troubles started. No one knew why. Shortly before he left to go into the antique business in Charleston he showed me the secret. It was his chewing tobacco, spat into the bath from time to time. From then on, one man in each shift chewed, and the problem was solved


What's the name of this book? It sounds really cool!

Any other recommendations on books about the history of MatSE/Chemistry?


The title is "Excuse me sir, would you like to buy a kilo of isopropyl bromide?"

https://archive.org/details/gergel_isopropyl_bromide


That's a hilarious book: "There was a jar which I had not noticed before containing potassium metal. I knew that potassium was a silvery metal, but this was one inch spheres, green with the oil in which they were immersed. I removed two for a collection of elements we were starting at Columbia High, scraped off the oil and put the marbles in my handkerchief which I added to a collection of miscellaneous glassware in my back pocket. "

Oops... I can see where that is headed.


Thanks for the rec! Gunna dive into it tonight!


Oh man, you are in for a treat! This was a terrific read even though I probably only understood 1/3rd of it.

Ignition! By John D. Clark

https://www.amazon.com/Ignition-audiobook/dp/B07CTW1M9D/ref=...

Or, maybe you’ve already read it, in which case: great job. Maybe someone else will see this comment and enjoy it.

I don’t want to spoil if for you, but I loved the last paragraph. I believe a new heroic age for the field is dawning.


Oh dang! This has been on my reading list for years! Definetly looking forward to checking it out!!!


My last chemistry class was in high school, so I had to glide over a lot of the details. But the story is still awesome even with liberal skimming of chem reaction details.

This book is probably helping me cope with the extreme confusion of all the various LK-99 attempts. Like, it's going to be a hugely messy confusion and slog with lots of results that are "blend-A can meet 5 of the 7 requirements, but not the important ones. blend-B can meet 3 of the 7, but one of the important ones. blend-C looks very promising but doesn't quite meet any of the requirements." for... years.


> Do you think it will be "purity" or understanding material variance/specific impurity?

It could be either.

As I pointed out in other comments, that's how radioactivity was discovered and it is very well possible that they blundered into something exceptional by accident, it is also possible that both parties got it wrong and there are yet other effects at play (see the big gap in the t/R curve, that really needs explaining).


According to wikipedia, that was observed by the original team in Korea.

https://en.wikipedia.org/wiki/LK-99#Replication_attempts

> Claimed to have synthesized LK-99 and to have measured superconductivity up to a temperature of 110 kelvin. Claimed to have observed an abrupt drop in resistance between ~300K and 220K, aligning with the Korean LKK team's results. Claimed to have confirmed structural consistency with x-ray diffraction.


Just a word of caution that Wikipedia entries around LK99 are unusually poor at this point in time. The citation of an attempt to publish it in Nature in 2020 is completely made up when the original Korean only mentions in passing a completely different paper that was withdrawn from Nature at the time. I hope the understandable excitement around this new invention will allow for a correct eventual accounting of the events.


Yes, but it does not drop nearly as far and there is the suggestion of equipment malfunction while at the same time that gear seems to work just fine around -110 C.


Bismuth is a strong diamagnet, and it can enter a superconducting state (at a fraction of a degree above zero), so it's not impossible. (And lots of materials are weak diamagnets and superconduct at some point.) So it wouldn't be unheard of. That said, LK-99 would definitely be an object of lots of research if it does have those properties.


I have seen it posted that diamagnetic materials require multiple magnets stuck together to create field "pockets" (?) for the material to rest in.

The few videos of lk99 show it reacting to a singular magnet. A property of superconductors that apparently diamagnetic materials don't have.


The term is “flux pinning”, and it only applies to the “quantum lock” effect. That is the specifically static hovering effect.

The diamagnetism, importantly this means repulsion of both poles simultaneously and equally (this is how you can have these magnets spin, a regular magnet repels same poles and attracts opposites, diamagnets repel both poles), is simply a characteristic of the superconductor, but it alone would just repel the object off.

Here is a timestamped link to NileRed’s YBCO video that visually describes the flux pinning:

https://m.youtube.com/watch?v=RS7gyZJg5nc&t=1887

And here’s a timestamped link to Ben Krasnow’s Applied Science YBCO video where he shows a close up of the crystal’s cross section that shows the imperfections that allow the magnetic field through for the pinning effect:

https://m.youtube.com/watch?v=sLFaa6RPJIU&t=75


Flux pinning of a superconductor should be able to hold it steady below a magnet, and the magnet should be able to drag the superconductor with it when moving (within reasonable weight limits of course). These will demonstrate for sure that levitation is not simply a force equilibrium between gravity, magnetic repulsion and one corner of the material resting on the surface.

There is a video from the Korean team showing LK99 moving when both poles of a large magnet is swung nearby, however the effect was a bit weak to conclusive.


Correct, the imperfections of the crystal allowing the flux pinning permits even a “hanging levitation”. Here is a timestamped video showing that:

https://m.youtube.com/watch?v=Ws6AAhTw7RA&t=90

If we develop methods of creating these superconductors with perfect crystal composition then there will only be the repulsion, allowing for levitation in a bowl shaped superconductor, but this “hanging levitation” would be impossible.

Perhaps we will develop manufacturing techniques to induce specific imperfections into the material to ensure predictable flux pinning; it seems like a useful, and wildly interesting side effect.


NileRed is youtube gold. It's one of very few channels that I follow and learn from.


I agree, great channel, but the cadence in his narration is atrocious.


It's fantastic compared to a lot of the people that I interact with on a daily basis so it doesn't bother me.


It only adds to the channel. I find it amusing.


When NileRed puts the YBCO on top of the row of bar magnets and it ping-pongs back and forth is awesome!

Timestamp: https://youtu.be/RS7gyZJg5nc?t=2496


The best thing about the NileRed superconductor video is it shows him initially failing to reproduce a YBCO superconductor after having already succeeded once before!

It goes to show how difficult manufacture, or in the case of the LK-99 news cycle “reproduction”, of these materials really is, and YBCO was a well documented area of superconductor manufacture.


And that he doesn't know what went wrong but comes up with a hypothesis and tests it and it works.


That's for levitating a diamagnetic material.

It'll react fine to a singular magnet, it just won't be stable enough to levitate - that's why the videos show casing replication of diamagnetism show it standing on end.


I would say any results that are already out are from people speedrunning production; give it another six or so days and there will be a ton of labs that have managed to produce various, better samples and will have performed decent experimentation on these.


The strength of the diamagnetism would be extremely anomalous if it were not a superconductor, and would need new physics/names - super-diamagnetism or something (which itself would still be very useful). The chances of it being new physical phenomena is vanishingly small - we're almost definitely seeing superconductivity at STP with this degree of diamagnetism.


Exactly. And from the various videos it doesn’t seem diamagnetic at all because it floats in a stationary way on a single planar magnet. Diamagnetic material can’t do that, they need an array of magnets.


The premise is flawed because that assumes the samples they are studying are identical.

Replication is difficult, and particularly difficult for novel processes where the important variables are not well understood. It could be that the methods were reported as accurately as possible but still leave out critical detail(s).


Interesting. Some people are getting strong diamagnetism at RTP, and now someone is actually seeing zero resistance at a low temperature.

I'd take this as mildly positive news. It would be a surprising material if it were a high Tc (only in the LN2 sense) superconductor that is also strongly diamagnetic at room temperature (though I'm not sure if that's more surprising than a RTP superconductor).


It's confusing. You'd expect both to be present or none, it is strange to have a material that is conducting and diamagnetic at room temperature and that shows super conductivity at much lower temperature. You'd expect the diamagnetism to disappear with the superconductivity.


Multiple teams have said they had to produce 10 batches just to get one sample with properties worth reporting on.

Perhaps the simplest explanation is that different teams are all ending up with different variations of a common material, with different impurities, crystal structure, etc.

There's likely a whole zoo of interesting materials here!


Yes, I thought that that might be the case from day one. When they wrote that they had only a 10% success rate it was clear that they were not at all in control of sample purity. The big remaining question for me is what happens when they start traveling with the original sample to have another lab test their samples and how those results compare with both the original results and the independently recreated different samples.

There is a good chance that there will be substantial differences between them.


“Purity” is a really overloaded term here. There are vast set of material properties that simply don’t map to a definition “purity” as in some homogeneous concentration of a material. This is so early no one likely knows exactly what configuration of material to “purify” for the intended outcome.

There will likely be years of not decades of looking at differences in the materials and performance of related materials to more fully explore this discovery.


Yes, true, let me unpack that then:

- purity as in the sample is uniformly constructed of the right atoms but they are not in the right configuration

vs

- purity as in the sample contains atoms that shouldn't be there in the first place

and finally

- purity as in: the sample that purportedly did show room temperature superconductivity turns out to be the impure one and that impurity is so poorly understood that we currently can not replicate it accurately, but a test by an independent lab of the sample would verify the properties as advertised.

All of these are possibles, and not mutually exclusive.


> purity as in the sample is uniformly constructed of the right atoms but they are not in the right configuration

The conclusions in the linked pdf suggest this may be the issue with LK99.

1. https://arxiv.org/abs/2307.16892


More than likely there is only one interesting material in this specific composition. But understanding the phenomenon we are observing in this material can potentially lead to the development of other materials which display these characteristics, and tune those to function in this way at specific temperature/pressure situations. If we develop efficient ways to produce these materials in bulk (which is orders of magnitude more complicated than just characterizing what we see here) it would be unimaginably revolutionary. But the energy required to do this at scale will likely require our civilization to utilize orders of magnitude more energy, so if this is practical for our daily lives on a wide scale I believe it's development will be contingent on harnessing fusion. Otherwise it will be limited to only the most extreme use cases in the way superconductors are currently used now.


My friend, what exactly do you think is so energy intensive with LK99 synthesis? I've briefly taken a look and the process proposed is really not that onerous in terms of energy consumed. It is a matter of perfecting the process that is the hurdle, we already spend tons of energy happily in similar industrial processes.


> But the energy required to do this at scale will likely require our civilization to utilize orders of magnitude more energy

As opposed to smelting aluminum or steel? And that creates stuff that is dirt cheap in bulk...


Observed effects which have been given a name != a single universal truth. Just because we have experimental evidence of effects and theoretical explanations for their occurrence prove we actually have defined the underlying phenomenon's which produce the observed effect accurately. The reason we experiment isn't to prove our theories are correct so much as it is to probe where they break down so we can further refine our understanding of the universe. What we are likely seeing here is the result of an organized pattern of atomic nucleii forming a lattice which creates the conditions for the movement of electrons to be easier (or harder) than expected at certain temperatures (with temperature being the property which induces strain on the lattice). This is very crude, but begins to explain the multitude of factors at play here.


Yes, that's true, but that would require a shift in our understanding of physics and everything about this saga so far has not done that. And that in part is a reason why people take this serious: it is believable. New physics would raise the bar considerably.

So for now I'm on the measuring error, impurity or process issue side of that without committing to which team I think has the problematic side.


The fact that our existing computational models are in agreement with the experimental behavior leads me to believe there is no new physics involved here. That being said this raises the question as to whether this class of material is known and characterized within our national research labs but has been kept classified, and if it was unknown it raises the question of why experimentation is what discovered it and why weren't we able to harness the computational model to identify it theoretically and develop a method to produce it practically prior to this. My money says there are people in this world who have long been aware of this, and its either already in use in a classified manner or it's interesting but there are other materials which are more practical in all use cases.


I think one factor that's overlooked with these simulations is that there are a ton of parameters to these which really hinder automated searching.

It's easy (relatively) to verify the results from a real world test since you know the physical parameters and can tweak the others based on intuition, where if the result matches the real world you can consider it valid, but if it doesn't you can have to check all sorts of things to be sure that it isn't a glitch due to some parameter not being reasonable.

That makes searching for materials really hard because you either need an absurd amount of computational power to be able to set the simulation parameters so high as to not worry about their effects or you get tons of false positives simply because the computer can't as easily tune those parameters to ensure it produces correct results.

As my PhD advisor has often put it regarding my own simulation work, if the simulations were that capable of modeling reality, there would be no need for billion dollar facilities to perform tests irl, you'd just spend all that on building many supercomputers.


> why we [we]ren't we able to harness the computational model to identify it theoretically and develop a method to produce it practically prior to this.

Because the computational requirements are off the scale in the most literal sense. The search space is so large that you won't be able to come up with an improvement in efficiency for your search unless you guide it very carefully with experimentally obtained results and that's exactly what these people were doing as far as I understand it. You mix up a batch of stuff, test it for gross properties, do crystallography and then use the information from that to do some numerical simulations to check if your assumptions and observations hold up.

I don't think we had this compound before.


If only part of the sample is superconducting and the other parts are ordinary conductors or isolators, the superconducting islands could show diamagnetism while being isolated from each other which means that eletrical current needs to pass resistive areas and therefore experiences resistance.


Could it not also mean an impure sample? Everyone is in a rush right now but none of these research teams are exactly experts of manufacture.


Just FYI, the wikipedia page for LK-99 has a very useful tracking grid of replication attempts with sources: https://en.wikipedia.org/wiki/LK-99


Can someone with field expertise explain the implications if this material were real / replicable / cheaply manufacturable? I see a lot of breathless excitement in these comments, as if free energy and perpetual motion machines are right around the corner, but from the few details I've delved into it's more like the electric grid might get 10% more efficient, MRIs might get a little cheaper, etc.

What are the exciting and apparently obvious applications that have everyone so excited? Is it a fusion / tokamak containment thing, that the cost of cooling current superconducting magnets is one of the big barriers to net energy generation?


Transmission loss is a real and important reason why you can't, say, cover the Sahara with solar panels to get unlimited free energy for mainland Europe and Africa (barring all the logistical challenges of actually pulling something like that off). This is because you lose a significant amount of energy just by moving it from one place to another.

With RTP superconductors, you get near perfect transmission from the site of energy production to the site of consumption. You could put wind turbine in remote sections of Montana and power up Chicago, something which previously would have been impossible.


Sahara solar panels were economically viable according to multiple studies, and there have been some limited plans made, but it has turned out politically non viable so far. Transmission losses are smaller than often assumed.


From what I've read, grid transmission losses are something like 5-10% - e.g. this page (https://www.eia.gov/tools/faqs/faq.php?id=105&t=3) cites 5% in the US. Fine and nice, but seems incremental, not earth shattering, and I wonder about the logistics / ROI of manufacturing thousands of miles of LK-99 cables to achieve that. But maybe I'm missing something.


At long distance, the transmission losses are so uneconomical – that they never got built in the firt place. This is why you shouldn't look at current loss numbers, which already weeded out what was not feasible before.

Now, if transmission losses are near-zero, then yeah – you'd still get the same power for the same price & losses (for other reasons), but from 500-1000 miles away instead of 50-100 miles. Residential customers won't notice anything immediately because they'll pay the same due to initial capital costs and stuff. But decades down the line it would slowly, invisibly transform everything around you.


The losses increase with distance, and decrease with higher voltages, which is why most cross country transmission happens at 740kVAC. But still, the energy you're using in your day to day life is almost always produced with 50-100miles of where it's consumed, because transporting it longer distances make losses uneconomical.


The BIG change is if this supports high magnetic flux densities, in which case literally everything that depends on magnetism or magnets or electromagnets gets an order of magnitude better - ten times as much power per unit of heat disappeared and ten times as much power per unit of mass or volume or just ten times more powerful. Remember how battery energy density increased by a factor of five (nicads -> lithium-ion) and we suddenly had quadcopters and flashlights that could burn paper at ten paces and handheld vacuum cleaners the size of a soda can that could suck pet hair out of shag carpets and cars running off batteries? Imagine that happening again.


But wait - modern electric motors, for example, are something like 80-95% efficient (https://en.wikipedia.org/wiki/Electric_motor#Efficiency), which implies there's not a ton of room to grow. Do superconductors somehow enable 10x more torque output for the same watt of electricity? Or is it that you could have a small motor in the palm of your hand that can consume huge amounts of electricity and produce enough torque to crack a diamond?


When the heat generation (inefficiency) is the main limiter in the technology, increasing the efficiency improves performance, rather than just saving power.

The electric motor limitations come from heat, so increased efficiency = increased strength. That 5-15% + work is what melts the motor when going beyond the rated load. I'm not familiar with the superconductors/electrical engineering but I think if no work is performed (motor is stalled), the motor will not get hot, basically just acting as a magnet.

From what I understand it is the same with computer chips, the biggest obstacle the chip companies have is heat generation; the chip gets too hot with the smaller designs. So less heat generation = faster chip.

Autonomous drones (for delivery and stuff) are extremely limited by the very low battery life. They fly around for 20 minutes and are done. Any complication and the battery runs out. So higher efficiency = longer battery life = more capabilities.

Nuclear fusion reactors also apparently benefit from higher temperature superconductors because it is hard to keep them so cold in a reactor. I know nothing about that though. To me it seems like 100 K or 300 K are very different from 100m K regardless but idk lol.


The latter. Motor losses are in large part due to resistance in the windings. Going from 95% efficiency to 99% efficiency means 1/5 as much heat output per unit of energy delivered, which means if your motor is thermally limited (which many are these days!) you can push five times as much power through it without it burning itself up. Similarly, increased achievable magnetic flux means a smaller motor can achieve greater torque and power so it can convert more electricity into movement.


They're far more efficient than a combustion engine, but electric motors and associated controllers/inverters do generate heat, and lots of it. DIY electric car conversion hobbyists struggle with this big time, for example. Cooling loops w/ pumps, fans etc. It's not as bad as the battery pack, but still a thing to design around.

I get the impression there's room for improvement esp in the inverter space.

Cars of course being only application of electric motors. I'm no electrical engineer, but I gotta think stuff like generators must have challenges?


Avatar 3 merchandise will definitely include floating rocks of Unobtanium.


For some reason what my brain keeps imagining is pulling out a drawer of a cabinet and it being perfectly silent and butter smooth because it uses mag-lev instead of wheels.

Mag-lev based bearings might be nice too.


That's already possible.


Transporting solar energy from deserts.


There are multiple mentions of "extraterrestrial super islands", is this a mistranslation of the Meissner effect? Great term nonetheless.


It's the mistranslation of room-temperature superconductivity.

"super islands" is likely translated from 超岛 which sounds the same as 超导 (superconductivity). I have no idea how 室温 (room-temperature) became extraterrestrial, must be extraordinarily bad speech to text model.

edit: could be 室温 (shi4wen1) -> shi4wai4 -> 室外(outdoors)/世外(out of this world) -> extraterrestrial


What a hilarious mistranslation. Somehow it is fitting though, as a totally unprecedented floating material.


You're exactly right on the extraterrestrial part: the speech to text transcribed it as "最近我们这个有关世外超岛的话题非常的火热"


I wish I could learn CJK. Will take a lifetime to master.


Yes, I saw that, there are some more obvious translation errors.


The extraterrestrial super island of stability


Let's break one rule at the time, please. I don't think I'm quite ready for heavy elements that are long lived, either at room temperature or any other, and even less so if they turn out to be non-radioactive...

https://en.wikipedia.org/wiki/Island_of_stability


The actual video is here: https://www.bilibili.com/video/BV1pM4y1p7u5

Short summary (not a physicist and consume most of my science in English, so may be slightly wrong):

- The material is fragile, and hard to form into a regular shape

- There's a weird drop in resistance around 230K; could be an artefact of the measurement process or instrument

- On 1 Aug they found a sample with almost 0 resistance, which got them really excited and led to them searching for more

- They tested 6 samples, only one of which exhibited zero resistance at below 110K

Basically, results are inconclusive, but it's promising.


This appears to be a great result.

"Room temperature" super conductor result? No.

But this is basically showing there is some superconductivity in the sample, and cooler temperatures expose intrinsic band structures.

Pretty exciting!


How is it exciting? It invalidates the original claim. We already have better SC.


It doesn't invalidate the original claim, as they haven't been able to make a pure sample to test with, as I understand it. Or at least that there is some difference between their samples and the others, but still it shows promise.


If they've made a novel superconductor, then that's pretty huge anyway, it opens new pathways for exploration, right?

Maybe it helps unlock the secret of what makes a superconductor superconduct, which could then be used to make other, more useful superconductors


[flagged]


> Purity does not affect Tc.

Here's a lump of 100% Cu and another lump of 50% Cu and 50% "other".

If purity doesn't matter, they must have the same properties? Why not just use 0.0001% copper in all our applications and save millions on material costs.


I know nothing about the theory of superconductors, but purity can affect electronic properties. For example, pure silicon is quite a poor conductor. If you dope it with a little bit of an n- or p-type dopant (an impurity), it conducts better. Add a bit more, and it conducts more. There are other effects, too.


Purity would affect superconduction. Consider grains of superconducting material embedded in a matrix of regular conductive metal.

One of the theories for the behavior of this material explains that the copper atoms preferentially form a structure that does not superconduct. The structure that does superconduct is tricky to achieve due to the energy levels. (Paraphrased summary)

It's likely that a poorly prepared sample will have discontinuous regions of both superconducting and non-superconducting material. If that is the case, you won't observe superconduction.

It may be that the non-superconducting material does superconduct at low temperatures, which would mask the purity problem.

The truth is that quite literally nobody fully understands what's actually happening here. That's kind of the point of all this experimentation


Holy shit what a statement.


The worlds best scientists don’t even know how to make room temp superconductors but this one person on HN is certain that purity doesn’t matter.


That supplemental video files and drama around submission process seemed to exhibit telltales signs of forged science, so the fact that this seems not nothing and is getting material scientists excited is exciting.


Ok not sure how to phrase this:

Could there be small pieces of RTSC inside the sample (causing the floating)

But then enough impurities to not be able to complete a circuit through the sample?

And cooling makes other parts of the sample super conducting?


Could be insulated RTP superconducting crystals, grains, islands with the bulk being a high temperature superconductor?


Yes exactly.


It seems likely. Given that the copper atoms prefer to form a non-superconducting material, it makes sense that there would be regions where the energy is just right to form the correct structure, and you get the incorrect structure everywhere else.

Given the way crystals grow, it then would follow that you'd get discontinuous crystals or grains within the material.

If we're taking bets, my money is on this exact thing happening. It'd explain the inconsistent results we've seen so far.


That's a failed repro then right?

Below 110K is below -163.15 Celcius

How would that compare to other superconductors?


It is and it isn't. It's at ambient pressure (which is something useful), and there is something very odd happening much higher up that needs to be explained. They say their sample purity is higher than the one the Korean team had, so that would normally lead to better yield and easier confirmation of the superconductivity. But since it does show the Meissner effect in other samples as well at room temperature there is a lot that still needs explaining before we can say it is a failed reproduction.


I adore stories that include the phrase "that's odd" or "something odd happens when ..."

Even if we don't get the astonishing result originally claimed by the rogue paper, it's still a triumph of science in my ignorant opinion.


And sometimes 'that's odd' leads to things larger than the original goal. You really don't want to hear those words in the doctors' office though.


perhaps an impurity caused the effect they're looking for


Yes, that's possible and something that has already happened once before: this is exactly how x-rays and eventually radioactivity were discovered, a chance contamination.


Tc = 110 K would take the #4 spot on https://en.wikipedia.org/wiki/List_of_superconductors


#3 spot at atmospheric pressure


At atmospheric pressure, no less.


> HgTlBaCaCuO

New band name. And band gap.


One of the most well known, YBCO, has a Tc of 95K.


Yeah it looks to me like either a replication failure or even evidence AGAINST a superconducting phase. A superconductor's resistance curve is supposed to show a sharp drop to zero at the transition temperature. If it's a dirty inhomogenous sample (e.g. specks of superconductor embedded in non-superconducting material), you get a kink where the curve descends to a non-zero background resistance. In the Southeast University data, there's a smooth curve that goes down until they get to the noise floor. There's no transition.


technically a failure, but still a strange result


I don't think it's a failure if the original claims are showing some kind of promise, science is being done, and the frontiers of knowledge are being pushed forward.

In other words, not "Eureka!" but "that's weird".


As I understand it, this is strong evidence of this being a novel new superconductor, but not at room temperature.

That's still TBD but hasn't been replicated by anyone reputable yet.


In the video, it says above 110K and not below, I got confused as well. I will add that a lot of people are deleting retweeting this. May be there is something more to it.


In the Resistance-Temperature graph (https://twitter.com/Lipez400/status/1686793608626663441/phot...) the resistance starts increasing from 110K


What’s the explanation for the sudden drop between 230K and 250K? It’s not dropping to zero but something is happening there.


Yes, very odd. This may be why the original team believes they have a superconductor on their hands, but it doesn't quite get there and yet it does show the Meissner effect so something doesn't quite add up yet.


Note that the team claiming a zero resistance replicate at 110K is the same team who said failed to replicate Meissner a few days ago. In the latest video they still don't think they have observed Meissner.


Yes, true, but other teams say they have so it's a bit of a mixed result.

But given some time all of that will resolve. Fascinating to see science at work at this level out in the open.


Original team had 20 years to play the synthesis lottery. Maybe you tweak the setting just enough and that low resistance drops to zero. Who knows, I’m still feeling optimistic.

He said in the video that their sample was more pure than the original paper. Would be wild if some impurity is what pushes it over the edge to a full blown super conductor.


They weren't quite playing the lottery as much as they were taking stacks of tickets and scratching them all off one by one to find something that looks like it might be a winner. This was very hard work. 100's or even thousands of samples.

> Would be wild if some impurity is what pushes it over the edge to a full blown super conductor.

I already mentioned this in another comment, but x-rays and radioactivity were discovered in that precise way.


It was exactly one of the point that were criticised in the original paper. I think that the supposed Reddit “expert” that ridiculed it should really be shamed to apologise to the authors. And together with him a lot of other people in various other places on the net.


perhaps it's an imperfect synthesis?


Or perhaps the other sample was an imperfect synthesis...


The first comment tells (from the professor talking in that video):

> In order not to misunderstand everyone, let me say that it is below 110K, and 0 resistance is observed at normal pressure.

Google translation, but I think it's clear that it's below and not above. At least normal pressure ... that's disappointing but still an improvement


I still don't understand what this is saying.

There was 0 resistance below 110 Kelvin (-165celcius)?

But not at room temperature?


In their sample they measured no resistance below 110 Kelvin. After 110 Kelvin resistance increased, although there was a weird dip in resistance around 225-250K that they can't explain (maybe instrument error.)


Yes.

At 110 the resistance is 0.0001 ohms. At the highest point on the chart, 200k (Still 100 below 0 F) the resistance is up to 0.1 ohm.


It should say "up to 110K".


Why would other teams be observing floating at room temp? Is it possible to have a diamagnet that becomes a superconductor?


It’s possible there are tiny bits of superconducting stuff at room temperature which connect into bigger and bigger bits as temperature drops, until it starts working as a whole at 110K.

Maybe. Don’t quote me.


That would suggest that a very high quality sample would superconduct at room temperature.


Unless the "low quality" part is actually causing the superconductivity. Remember the multiple simulations that showed Cu replacing Pb at a less favorable site (energetically) was better for superconductivity than at a more favorable site. It's possible "high quality" samples actually have less "imperfections", which seem important for the superconductivity.


Exactly it's quite possible that 'worse' is actually 'better' and vv. I'm sure that it won't be long before that will be resolved though, this is all happening so fast that such important answers will be sought by more than one team.


Not necessarily, but possibly.


Due to how these materials behave, this likely means you can manufacture a real-deal UFO in orbit…


what? give me an effective field theory for that lol because that is not a thing



The one interesting thing I should note is that synthesis of the copper phosphide intermediate tends to result in a copper deficient product, so I wonder if the presence of lattice defects in this might have an effect on where the copper atoms intercalate in the lattice.


In American dollars that is -261.67F, a little chilly for room temp.


But on the warm side for a superconductor, and displaying some weird behavior just below water ice temperatures.


One point in favor of a novel superconductor material, one strike against it being a superconductor at room temperature.


Why are all these materials confirmations coming from Chinese speaking scientists? Do scientists in the US not do materials science so much?


I'd expect DuPont, BASF, Apple, Google, Intel, IBM, GE, Kodak if any of their labs remain, any number of biotech labs like Genentech might have that are sufficiently mat-sci adjacent, all of the major of universities; MIT, Harvard, etc. Any of the national labs. Sandia, Lawrence Livermore, whichever are appropriate. It's just that whole DEA and red phosphorus being a meth precursor making it difficult. Yay, war on drugs.


US does fine; likely in less of a rush to be first-past-the-post on announcing results. When the big US labs announce, they will likely be more careful and vetted and authoritative.


I guess that you can thank the DEA for this specific case…


What about European or Japanese scientists? And about Korean scientists not part of the original team?

The last should also be able to get their hands on a sample provided by the original team and won't have to do reproduction attempts, right?


Who told you that they are not attempting the reproduction? Even US labs are attempting it, some of them explicitly in the open like the Argonne National Laboratory. Here you can find a non-comprehensive list of reproduction attempts: https://forums.spacebattles.com/threads/claims-of-room-tempe...


Oh, so there are finally Korean labs in that list! Last time there were 1 American, 1 Indian and 5 Chinese labs which I thought very weird. Hopefully the Korean labs can get an original sample.


The Drug Enforcement Administration? What does that have to do with physics?


This is probably about the fact that red phosphorus (required for the original process) cannot be purchased without a license (https://nitter.net/andrewmccalip/status/1684191067477004288#...), so kitchen chemists are out of the race. Established labs should have less problems though.


Plot twist - a cartel holds a press conference, and beats everybody to the punch


Because the US/EU labs are being methodological and careful. This is by far the most reputable result to come out of China so far on LK-99


996 culture


The Fig 3a)b) graphs are not showing superconductivity. That just looks like the resistance dips below the ~1e-5 mark and goes goes deep into the noise. That just looks like a normal conductor that has a positive temperature coefficient.

edit: to add speculation, I think they made something that's not quite LK-99, and it's behaving like a normal conductor with no superconductance.


An excitation of 1mA and R = 1e-5 means the voltage is 10 nV (1e-8). Also the chart has a logarithmic y-axis.


I know, I'm just saying their test condition has a really high noise floor and once you dive into the noise floor there is no meaningful result.


That would be my interpretation too. Usually you get a stronger transition to 0 at the critical temperature but I can't see this (although it could be hidden under the noise)


The noise is probably just an artefact from measurement.


that's almost literally the definition of noise.

I'm saying the results are useless because the noise floor is obscuring any interesting behavior (if any).

Some people are saying "well, the Tc is 160K, so this result is invalid"--the way I see it, the Tc is not 160K because the test setup is so noisy so you're not really seeing any superconductivity at any temperature (not because it doesn't exist, but because their test setup is shitty)


I also agree, looks like they just reached the instrument noise floor. This result should be taken with a grain of salt.


What would be the acceptable noise floor here?


that's the wrong way to look at this.

what i see in this noisy measurement is nothing interesting. that always means:

a) the noise is obscuring something interesting

b) there is nothing interesting

for a), if it's practical and/or likely that there is something interesting in the noise, I'd try to find a way to lower the noise (or SNR).

you would assert b) if you have some strong convictions that there's nothing interesting down there.

Since the assertion that LK-99 is superconducting at room temp, there's already enough data to say that this is either A) whatever they measured is not LK-99 B) whatever they measured is LK-99, but doesn't superconduct at room temp.

Their ridiculous interpretation pointing at the SNR=1 point saying that's the critical temperature is actually hilarious.


Read the graph, there is a sharp drop in resistance from 220K-250K. Its still not superconducting but it could be hinting something?


Could indicate some superconducting regions in the whole sample, or some weird anisotropic superconduction at higher temps...

That sharp drop, if not equipment error, could be a lot of things.


That’s what I was thinking. In the video he brushes it off as a potential equipment malfunction, but perhaps it’s the effect everyone is looking for.


A sudden discontinuity like that but continuing the same curve and jumping back... that's equipment malfunction 99% of the time.


And yet, they published that instead of figuring out what is possibly wrong with their gear which in turn may invalidate the rest of the their results. I'm not going to second guess their motivations though, they probably know what they are doing but it is interesting and deserves explanation.


Seems reasonable, then again we're dealing with a potential superconductor where sudden discontinuity is the expected result.


Why publish this graph without running the measurements again though? Serious question since I’ve no idea about the effort needed to get this data.

I’ve got a small hope that they actually did and found the effect didn’t go away. They’ll still say ’equipment malfunction’, there isn’t any downside, only upside if it gets reproduced somewhere else.


I'm really curious what the eventual explanation of this is and whether other teams will observe this happening as well.


Almost all replication attempts are done on Bilibili, which namesake Mikoto Misaka using Railgun which would be made by superconductors.

Very beautiful.


Novice question - does a material behaving as a superconductor occur all the sudden like a step function if resistance is plotted against temp and pressure - or does it ease into super conductor behavior?

(edit) Also is it resistance (aka if I remember my EE degree correctly), DC only or also impedance where frequency also matters?


It's like freezing water.

It happens at a single temperature. It doesn't happen at once, there is a small amount of heat you must take from the material before it becomes a superconductor. Regions of it become superconductors on the process, and those grow until the entire material change.

After the change, it becomes a stronger (yes, there is such a thing) superconductor the cooler it gets.

(Or, at least that's what happens to the kinds I know about. The thing is complex enough that I wouldn't be too surprised to learn about one that behaves differently.)


>After the change, it becomes a stronger (yes, there is such a thing) superconductor the cooler it gets

Do you mean the tradeoff between temperature, field strength and current? Like, if you lower the temperature, the SC will be able to handle a stronger field or more current in return?


Yes.


I do find that interesting... I found a graph of another HTSC's (YBCO) resistance vs temperature [1], and the function is much more discontinuous looking than the one shown in the article. That would lead me (someone not at all trained in the physics of superconductors) to believe that perhaps the purity of the sample is a factor here, with different parts of the sample having different critical temperatures, gradually decreasing resistance until an inflection point is reached at 110K.

1. https://www.researchgate.net/figure/The-electrical-resistivi...


The ResearchGate chart has a linearly scaled y-axis, the Southeast University chart has a logarithmic scaled y-axis.


all at once. It instantaneously expels the magnetic field. You can observe experiments on youtube: https://www.youtube.com/watch?v=RS7gyZJg5nc#t=32m30s


Only type-I superconductors expel significant magnetic fields. Virtually everything you see on YouTube is a type-II superconductor. type-I superconductors require liquid helium.

Also, only type-II superconductors can hover in a stable configuration near magnets. Flux pinning is required for that.


> (edit) Also is it resistance (aka if I remember my EE degree correctly), DC only or also impedance where frequency also matters?

Just resistance. Impedance in an ideal BCS superconductor goes roughly as the square of the AC frequency.


The fact that result seem to be all over the place has me hopeful.

If it was just straight up wrong/fraud it wouldn't be playing out like this.

Perhaps with some finetuning we'll nail it down.


> below 110 K

It is not room temperature, is it?


Room temp is 300K. Liquid nitrogen boils at 77K, though.


I suppose it is, if the room is in Mars.


Not Canadian, eh?


0 C is 273.15K, so room temperature is around 295 K. 300 K is a decent approximation to make back-of-the-envelope calculations easier, but I don't know anybody that actually keeps their room at 80 F.


Fascinating and ultimately confusing result. What is happening in that higher temperature window where the resistance suddenly drops orders of magnitude?


Incredibly poor showing on HN around these issues. Lay people vastly over estimate their ability to reason/comment on physics issues.


I want to believe.


The truth is out there.


There’s a reason USO is a pun in Japanese.


Looks like it still goes superconducting at a magnetic field of 9 Tesla. Does that seem like a really high critical field?


YBCO which has a critical temperature of around 90K has a critical field in excess of 250T. So 9T is not expected to weaken superconductivity much for a sample with a critical temperature of 110K.


Wow 250T is pretty high. Was that measured or extrapolated?


Extrapolated.


The linked video mentions something about "outer space islands" in the subtitles. Is that just bad auto-translation or did they seriously discover something amazing?


They found “islands” of superconductivity on the resistivity graph.


see this other comment[0] its most likely a translation issue

[0] https://news.ycombinator.com/item?id=36975492


I'm curious why the resistance is dropping by a lot in figure 3 a) between around 230 and 260 K


The jump looks like bad contacts to me. The wires you attach undergo stress during the cooldown, and such artefacts can happen with small samples. Rewiring the sample should get rid of it.


This doesn't look superconducting at all to me, simply the sample is very small and below 110K the resistance of the sample is too small to be measured by their measurement equipment. I'm happy to be proved wrong but there should be a steeper transition at the critical temperature.


Note that the graphic is in log escale.

Also, it looks like it's more complicated https://www.researchgate.net/figure/Superconducting-transiti...

Also, IIUC the method makes a lot of small grains with different composition, and they may become superconductors at different temperature, and make a dirtier curve.

Also, not my research area, so I'm guessing a bit.


Shouldn't the title say "above 110 K" ?


This so far is saying this stuff isn’t room temp superconductor, but a high temp superconductor which means the paper is mistitled and didn’t deserve all the hype


good not great.


TOTK Sky Islands effect is what we're looking for.


credibility of LK99 claims is very very high. What are the medium term practical implications for all these potential applications?


It seems more and more like it's credible, but that synthesis is going to prove to be the issue. All these repro attempts are having too much success for there to be nothing behind the team's claims.


We also don't know its critical field strength. The original measurement from Korea was low. That could improve as they improve synthesis, but could be problem with the material. There are lots of high temperature superconductors that aren't useful; YBCO is important because of high field strength and liquid nitrogen coooling.

I'm sure there are lots of uses for low current room-temperature superconductor. But powerful magnets and long-distance power transmission require large currents and big magnetic fields.


Can you explain why -160c is a measure of success when the claim was room temperature? A super conductor functioning at -160c would make MRIs simpler, but it's not world changing.


because it's a very unexpected result, it shouldn't be levitating at room temperature and the random drop in resistance at certain temperatures doesn't make sense


Does anyone know what S.R. Hadden has been up to lately? A new high-temperature superconductor seems like the perfect cover story for a fraud scheme around room temperature superconductors.


I don't think -160C would change much since it would still need cryogenic cooling. Liquid nitrogen is the most common and cheapest cryogenic. YBCO is liquid nitrogen cooled and has the advantage that can make in large quantities and has high field strengths.

But if it was slightly warmer, over -153C, then it could use non-cryogenic refrigerants.


Credible? Their claim was “room-temperature superconductivity.”


It looks like they've at least discovered a novel high temperature (relatively speaking), low pressure superconductor. That does lend some credibility to the original claim, and perhaps reproduction of the result is trickier than originally thought.


There’s a Grand Canyon sized chasm between creating a “low pressure superconductor” and creating a room-temperature superconductor and, unfortunately, Evel Knievel wasn’t a co-author.


Start shorting power companies.


Hypothetically, if we did end up with a worldwide superconducting energy grid, this would smooth load, which I would expect would make generation cheaper (since we can get rid of the need for expensive peaker plants and/or stationary storage requirements to handle local demand spikes). This would therefore make power companies more profitable, no?


Probably depends on the power company.

The ones with a lot of coal plans would suffer; the ones with a bunch of solar/wind farms in prime locations that can dramatically expand capacity (think things like giant solar farms in the Australian outback) would benefit.


HVDC and HVAC transmission lines already only see single digit transmission losses over hundreds of miles

This has bigger implications in reducing wiring (and weight) costs in things like electric vehicles. Instead of fat finger diameter 20' copper cables, you could replace them with tooth floss.


> HVDC and HVAC transmission lines already only see single digit transmission losses over hundreds of miles

Quite a bit of the world is thousands of miles from sunlight at any particular time. Being able to power Northern Europe off solar farms in the Sahara has the potential to fix a number of challenges with green energy.


Morocco has been working on this plan for ages. Coast to coast (~3000 miles) in the US is much less than 9%. It's possible with current technology, just nobody has the inclination to execute right now.



upvoted for the latent pun


0 applications out of the door but huge leap for material science. And some of the best and brightest people will look for something deep there.


What do you mean? If it's only a superconductor below 110K, that's not nearly as significant.


Yes, that's what one team now says: that they have observed super conductivity and that this happens at a much lower temperature than the one claimed by the other team. But it is at ambient pressure and more importantly there are already multiple confirmations of the Meissner effect at room temperature. Besides that this experiment shows some really weird stuff happening at higher temperatures that needs to be explained.

So this race is far from run yet.


What confirmations?


Very informal "leaked" confirmations, but that is the current state:

https://twitter.com/elsa17z/status/1686763798294593536?s=20


increased geopolitical tension


That is an interesting observation and puts words to some thoughts that I've had: if it turns out that this is 'the real thing' and the patent holds up South Korea is suddenly a superpower.


Depends. This feels a bit like the Wright Brothers patenting wing warping. It's possible that innovation will happen so quickly that the original patents become worthless.

Frankly there is also zero chance that the patents are respected by all parties (i.e. China), regardless.


Yes, that is a definite possibility.


IP laws--should any even apply--won't prevent any state from using this material should the claims be proven true.


Of course it won't. But then I predict that there will be a massive impact on IP laws as a construct. Because this is the one that counts, if they don't work here they're doomed.


If LK-99 turns out be a legit room temp superconductor I can't see traditional patent/IP rules applying to it. The value to our species to too high for a technology like that to be encumbered. I would think a very large lump sum payment to the patent holders would be sufficient to cover everyone's interest.

Otherwise I fully expect every government in the world to simply ignore the patent and allow public/private use -- effectively invalidating the patent.

There is existing precedent for this type of action: the US government seizing wireless patents during WW1 or the Indian government invalidating international patents on medication for the purpose of federal manufacturing.


That doesn't follow at all. Lead and copper aren't exclusive to South Korea. The authors get prestige and the owners of whatever patents are granted get some money.


Of course they aren't. But patent owners get to set the terms under which their patents are licensed. It's not like the record business where there is a fixed deal and if you use someone's lyrics you know up front what it is going to cost you. Someone might not even want to do business with you at all...


> But patent owners get to set the terms under which their patents are licensed

To private entities, sure. The US government can and will force you to grant them a license if necessary.


Yes, except that's the US government and the inventors here are in South Korea. I don't think the US would get away with declaring 'eminent domain' over something invented in a different country. They could choose to simply not honor the patent but that will open a massive can of worms, especially because a lot of this stuff depends on reciprocity: if your government doesn't honor our key patents, why should we do the reverse?


Carbon, oxygen, hydrogen, nitrogen, phosphorus and sulfur make up the bulk of organic molecules and plenty of those are covered by world wide patents (drugs).


I don't think you can patent a material, just the methods you use to produce it. And if this is the real thing, there are probably dramatically more efficient ways to produce than this first step.


I don't think you should be able to, but there is this story that I read a while ago that makes it seem like you can:

https://www.nature.com/articles/35003008




Guidelines | FAQ | Lists | API | Security | Legal | Apply to YC | Contact

Search: